Endocrine/DM/Thyroid/Gastro-Hepatic/GU/OBGYN/Family Planning/Peds

Lakukan tugas rumah & ujian kamu dengan baik sekarang menggunakan Quizwiz!

Normal AST level, indication?

0-45 not liver specific, can be affected by other conditions (AMI)

Normal TSH level

0.5-5.0

Lab results hyperthryoidism

<0.5 tsh, high T4 high T3

Lab results subclinical hypothyroidism

>5.0, normal T4 normal T3

Criteria Dx DM

A1C >6.5% FBG 126 or more Sx hyperglycemia + random BG >200 2 hr glucose > 200

Definition of Anti-HCV

Antibody hepatitis C Virus screening test for Hep C 85% will become carriers If positive test order HCv RNA or HCV polymerase chain reaction to rule out chronis infection

How to differentiate UC vs CD

CD will have formation of fistula/anal disease UC have hematochezia (bloody mucus diarrhea)

Visual acuity testing with an eye chart should begin at what age? 2 Years 3 Years 5 Years 7 Years

Correct answer is: 3 years. Vision screening is an efficient and cost-effective method to identify children with visual impairment or eye conditions that are likely to lead to visual impairment. The method used to screen a child will depend on the child's age, but screening should be performed periodically throughout childhood. Visual acuity testing with an eye chart requires a cooperative child, so testing is most successful with children 3 years and older. If a child is unable to cooperate for visual acuity testing at age 3 years, a second attempt should be made within 6 months.

Match each type of emergency contraception with the FDA-approved maximum amount of time post-coitus that it can be effectively used. A. 24 hours B. 3 days C. 5 days D. 7 days 1. Levonorgestrel __ 2. Ulipristal __ Levonorgestrel-A Ulipristal-B Levonorgestrel-B Ulipristal-C Levonorgestrel-C Ulipristal-A Levonorgestrel-B Ulipristal-D

Correct answer : Levonorgestrel- 3 days; Ulipristal- 5 days. When administered properly, emergency hormonal contraception reduces the risk of pregnancy by ≥89%. Levonorgestrel is approved for use up to 3 days (72 hours) following unprotected coitus, while ulipristal is approved for up to 5 days (120 hours) post unprotected coitus. Both options have similar effectiveness when taken within 3 days after unprotected intercourse, but ulipristal has been shown to be more effective than levonorgestrel when taken between 3 and 5 days after unprotected intercourse.

When evaluating the efficacy of estradiol (Vagifem) vaginal tablets, the nurse practitioner expects that the patient will say: "I have a bit of vaginal dryness but otherwise I'm OK." "I have a bit of a red vaginal discharge." "Sexual activity is more comfortable." "I'm urinating less frequently."

Correct answer is: "Sexual activity is more comfortable." The purpose of vaginal estradiol is to locally replace the estrogen that is lost as a function of menopause. Replacing vaginal estrogen helps to reestablish the lactobacilli population and supports a more lubricated and premenopausal environment. As a result, the clinician expects to hear from the patient that intercourse is more comfortable. Vaginal estradiol should not produce urinary discoloration or impact urination

In order to remove a drug with an elimination half-life of 3 hours from breast milk via "pump and dump," the process must be continued for at least: 3 hours 9 hours 30 hours 60 hours

Correct answer is: 9 hours. The primary determinant of drug concentration in breast milk is the drug concentration in the mother's plasma. After 5 elimination half-lives (T½), a drug will be approximately 98% eliminated from maternal plasma, and this does provide a predictable frame of reference. Therefore, for breastfeeding mothers who wish to "pump and dump," 3-5 elimination half-lives are appropriate. As all drugs have different T½, a fixed time period such as two infant feeding cycles or approximately 8 hours is not appropriate

You are teaching a 62-year-old patient with type 2 diabetes mellitus about using mealtime insulin to help with the management of post-prandial hyperglycemia. You describe a situation where blood sugar is between 200 and 250 mg/dL (11.1‒13.9 mmol/L) and advise the appropriate dose of rapid-acting insulin to attain a goal of <150 mg/dL (8.3 mmol/L) is: 1 unit. 2 units. 4 units. 8 units.

Correct answer is: 2 units. The American Diabetes Association (ADA) endorses the use of mealtime insulin to achieve glycemic control in patients whose A1c remains >6.5% after 3 months of optimal oral therapy. One unit of rapid-acting insulin will result in a blood sugar decrease of approximately 50 mg/dL (2.78 mmol/L). Therefore, in the example above, 2 units will be needed to attain the goal of <150 mg/dL.

In which of the following situations is parental consent usually needed prior to treatment? A 16-year-old requesting information about contraception A 17-year-old who wants help with smoking cessation A 16-year-old requesting treatment for acne vulgaris A 17-year-old who requests treatment for a sexually transmitted infection

Correct answer is: A 16-year-old requesting treatment for acne vulgaris. All 50 states have legislation that protects adolescents to ensure confidentiality for treatment related to reproductive health and mental health issues. Otherwise, parental consent is required for treatment in children under the age of 18

Which of the following would you most likely recommend for follow-up imaging following resolution of their UTI? A 24-year-old nonpregnant woman with acute, uncomplicated UTI A 36-year-old nonpregnant woman with 2 acute, uncomplicated UTIs in the past 9 months A 54-year-old man with acute bacterial prostatitis A 42-year-old man with type 2 diabetes mellitus and recurrent pyelonephritis

Correct answer is: A 42-year-old man with type 2 diabetes mellitus and recurrent pyelonephritis. A complicated UTI has the following four aspects: structural abnormalities, metabolic/hormonal abnormalities (e.g., diabetes or pregnancy), impaired host responses, or unusual pathogens (e.g., yeasts). Patients with a complicated UTI can be considered for abdominal ultrasound to identify abnormalities that can predispose the patient to further infections. Among those listed, the man with type 2 diabetes mellitus and recurrent pyelonephritis would best qualify for diagnostic imaging.

Which of the following women would least likely be an acceptable candidate for post-menopausal hormone therapy? A 54-year-old taking antiepileptic medication for seizure disorder treatment A 48-year-old with a BMI=28 kg/m2 who is physically inactive A 45-year-old with well-controlled hypertension and taking an ACE inhibitor with a thiazide diuretic A 47-year-old who is a former smoker with a 20 pack-year history

Correct answer is: A 54-year-old taking antiepileptic medication for seizure disorder treatment. Contraindications on the use of hormone therapy in postmenopausal women include unexplained vaginal bleeding, acute liver disease, chronic impaired liver function, thrombotic disease, neuro-ophthalmologic vascular disease, and endometrial or breast cancer. HT should be used with caution in women with seizure disorders (due to the potential for drug-drug interaction) or dyslipidemia. HT has been linked to cardiovascular disease and thus the risks/benefits of HT in women with pre-existing risk factors for heart disease should be carefully discussed before initiating therapy. Given the risk of cardiovascular disease with HT, it is generally not recommended to initiate HT in healthy older women after menopause

Which of the following infections would qualify for 7 days of oral antimicrobial therapy? A 34-year-old nonpregnant woman with an acute UTI receiving TMP-SMX A 42-year-old nonpregnant woman with an acute UTI receiving fosfomycin A 57-year-old man with a UTI receiving ciprofloxacin A 52-year-old man with chronic bacterial prostatitis receiving levofloxacin

Correct answer is: A 57-year-old man with a UTI receiving ciprofloxacin. Absent local geographic resistance, uncomplicated lower urinary tract infection in well women should be treated with a 3-day course of antimicrobial therapy. Alternatively, a 1-day regimen of fosfomycin can be used. Longer regimens are indicated in men, pregnant women, or those patients with other risk for poor outcomes (e.g., diabetes mellitus, age >65 years). A 7-day course with a fluoroquinolone is recommended for UTIs in men. Despite concerns of fluoroquinolone resistance, the benefits of ciprofloxacin treatment outweigh the risk for this indication. Chronic bacterial prostatitis is typically treated for 4 weeks with a fluoroquinolone

A 47-year-old man presents with a chief complaint of perineal pain with defecation and is diagnosed with acute bacterial prostatitis. He has had one sexual partner for the past 15 years. Which of the following statements would be most accurate for this patient? Chlamydial infection is likely. An oral macrolide or doxycycline are first-line treatments. A Gram-negative organism is the most likely cause. A 3-day course of antibiotic therapy is preferred

Correct answer is: A Gram-negative organism is the most likely cause. Acute bacterial prostatitis, regardless of causative organism, is frequently characterized by perineal pain with defecation and this complaint significantly raises the index of suspicion. Causative organisms are generally presumed to be sexually transmitted, e.g., gonorrhea or chlamydia in patients <35 years of age and Gram-negative intestinal flora in those >35 years of age or in men with low risk for STI. Men who have sex with men are also at higher risk of Gram-negative urinary tract infection, regardless of age. Antibiotic therapy for a minimum of 10 days (and up to 4 weeks) is recommended to ensure adequate penetration in the prostate gland

A 9-year-old child with moderate persistent asthma is about to receive injectable influenza vaccine. His parents and he should be advised about all of the following except: The vaccine is typically 70%-90% effective in preventing influenza The vaccine can be administered during antibiotic therapy A short flu-like syndrome typically occurs after immunization Localized immunization reactions are fairly common

Correct answer is: A short flu-like syndrome typically occurs after immunization. The injectable influenza vaccine is an inactive virus that can cause muscle soreness or erythema at the site of injection. Patient factors create an appreciable influence on vaccine effectiveness, and estimates range from 70%-90%. The vaccine can be used with antimicrobial therapy, as can any vaccine licensed in the US. The vaccine does not cause a significant flu-like syndrome. That did happen decades ago when the vaccine was reconstituted in horse sera, but it does not happen with contemporary influenza vaccines.

Which of the following is consistent with the diagnosis of diabetes mellitus? Fasting plasma glucose=100-125 mg/dL (5.6-6.9 mmol/L) A1c >6.5% Plasma glucose=140-199 mg/dL (7.8-11.0 mmol/L) on the 75-g oral glucose tolerance test Random plasma glucose ›125 mg/dL without classic diabetes mellitus symptoms

Correct answer is: A1c >6.5%. Diagnostic criteria for diabetes mellitus include (1) fasting plasma glucose >126 mg/dL on two occasions, (2) A1c >6.5%, (3) plasma glucose tolerance >200 mg/dL 2 hours after a 75-g glucose load, or (4) a random plasma glucose >200 mg/dL along with classic symptoms of polyuria, polydipsia, or polyphagia along with unexplained weight loss or hyperglycemic crisis.

The parents of a healthy term newborn present their infant for the initial well child visit. The infant is being formula-fed, and the parents ask how much should she be eating. You respond: About 1-2 ounces (30-60 mL) every 3‒4 hours About 1-2 ounces (30-60 mL) every 6 hours About 2-3 ounces (60-90 mL) every 3‒4 hours About 4-6 ounces (120-180 mL) every 2-3 hours

Correct answer is: About 2-3 ounces (60-90 mL) every 3‒4 hours. A healthy term newborn who is formula-fed will generally take 2-3 ounces (60-90 mL) per feeding and will feed every 3-4 hours during the first few weeks, with some babies feeding more often in smaller amounts. Breastfed newborns tend to take smaller, more frequent feedings than formula-fed infants. By the end of the first month, the infant should be up to 4 ounces (120 mL) per feeding, with a fairly predictable feeding schedule of about every 4 hours.

The new mother of a neonate is HBsAg positive. She wants to breastfeed her baby. The most appropriate action is to: Administer hepatitis B immunization and hepatitis B immune globulin to the child Administer hepatitis B immunization to the mother Administer hepatitis B immunization to the child Administer hepatitis B immune globulin to the child

Correct answer is: Administer hepatitis B immunization and hepatitis B immune globulin to the child. The presence of HBsAg indicates that the mother is currently infected with hepatitis B. Therefore, the neonate should be given hepatitis B immune globulin to confer immediate protection and allow the mother to breastfeed. Hepatitis B immunization should be given in accordance with typical childhood vaccination schedules to ensure that the child develops long-term active immunity

According to the US Medical Eligibility Criteria for Contraceptive Use, which of the following clinical scenarios describes a Category 3 (theoretical risks usually outweigh the advantages) situation for the use of combined oral contraceptives? Factor V Leiden mutation Age ≥35 years and smokes <15 cigarettes per day Varicose veins BMI ≥30 kg/m2

Correct answer is: Age ≥35 years and smokes <15 cigarettes per day. Age ≥35 years and smokes <15 cigarettes per day is a Category 3 circumstance, in which theoretical risks generally outweigh the advantages. Factor 5 Leiden mutation, a genetically-based thrombophilia, is a Category 4 condition, in which use represents unacceptable risk. A BMI ≥30 kg/m2 is Category 2, in which advantages outweigh the risk, and varicose veins are Category 1, in which there is no identified risk

You see a 62-year-old man with benign prostatic hyperplasia (BPH) and who is being treated for hypertension, heart failure, and depression. Which of the following medications is most likely to cause acute urinary retention in this patient? Lisinopril Amitriptyline Propranolol Thiazide diuretic

Correct answer is: Amitriptyline. Amitriptyline is a heterocyclic antidepressant that has significant anticholinergic properties. Anticholinergic effects include relaxation of smooth muscles, including the smooth muscle of the bladder. In this setting of heightened relaxation, bladder contraction is inhibited. Impaired bladder contraction combined with the mechanical obstruction to urinary outflow that an enlarged prostate can create can produce significant urinary retention.

Which of the following is the most appropriate antibiotic for a 7-week-old with acute otitis media? Cephalexin Trimethoprim-sulfamethoxazole Erythromycin with sulfamethoxazole Amoxicillin-clavulanate

Correct answer is: Amoxicillin-clavulanate. An infant under the age of 6 months with acute otitis media (AOM) should be treated aggressively with antibiotic therapy. Amoxicillin-clavulanate will cover drug-resistant Streptococcus pneumoniae, beta-lactam‒susceptible and beta-lactam‒resistant Haemophilus influenzae, and Moraxella catarrhalis, the three major bacterial pathogens causing AOM, minimizing the risk of treatment failure.

Which of the following is a first-line therapy for the treatment of acute bacterial rhinosinusitis in a 9-year-old boy with no risk for antimicrobial resistance and no known drug allergies?

Correct answer is: Amoxicillin-clavulanate. Common pathogens that cause acute bacterial rhinosinusitis (ABRS) in children include S. pneumoniae, H. influenzae, and M. catarrhalis. According to guidelines from the Infectious Diseases Society of America (IDSA), first-line treatment of acute bacterial rhinosinusitis in children with no risk for antimicrobial resistance is amoxicillin-clavulanate. For patients with a beta-lactam allergy, first-line treatment can include levofloxacin monotherapy or clindamycin plus cefixime or cefpodoxime

Which of the following is a prudent treatment choice for an otherwise well 1½-year-old with a 4-day history of mild-to-moderate otalgia with intermittent fever to maximum of 101.1⁰F (38.3⁰C) with no known allergies who has acute otitis media and is in need of antimicrobial therapy? Cefixime Amoxicillin Trimethoprim-sulfamethoxazole Azithromycin

Correct answer is: Amoxicillin. The primary treatment target in acute otitis media is Streptococcus pneumoniae. Absent severe otalgia or a temperature >102.2⁰F (39.0⁰C), it is not necessary to cover beta-lactam-resistant strains or atypical pathogens. Consequently, amoxicillin is the preferred drug of choice in this scenario

Which of the following is most consistent with ADHD? A 14-year-old boy whose school performance has decreased dramatically in the past year An 8-year-old boy who does not seem to listen when spoken to at school and at home A 4-year-old girl who constantly squirms in her seat at church An 11-year-old boy who does well in school but fails to finish chores at home

Correct answer is: An 8-year-old boy who does not seem to listen when spoken to at school and at home. The key diagnostic components of ADHD include symptoms that are present before age 12 years, impairment is present in at least 2 settings, and there must be evidence of functional interference, either socially, academically, or in extracurricular activities. Of the children listed, the 8-year-old is most consistent with ADHD, though other criteria would need to be fulfilled for a diagnosis. These criteria can vary depending on the type of ADHD (i.e., inattentive, hyperactive/impulsive, or combined)

The NP sees a 27-year-old mother who is breastfeeding a 4-month-old infant. The mother is diagnosed with acute bacterial rhinosinusitis and requires antimicrobial therapy. Which of the following is the least appropriate choice? Amoxicillin Amoxicillin-clavulanate Cefixime Doxycyclin

Correct answer is: Doxycycline. Dr. Thomas Hale has published a 5-level Lactation Risk Category Classification System for medications ranging from L1 (safest) to L5 (contraindicated). Amoxicillin with or without clavulanate is Category L1, while cefixime is Category L2. Doxycycline is Category L3-L4, as chronic use has been associated with teeth color changes and lessened bone growth

Which of the following is the best candidate to receive varicella zoster immune globulin (VZIG)? A healthy vaccinated 4-year-old who will be traveling to an area where chickenpox is endemic An unvaccinated 3-year-old that will be starting daycare An unvaccinated 8-month-old with a chronic immunocompromising condition who came into close contact with an individual with active chickenpox A healthy 12-year-old with a history of chickenpox and who came into close contact with an individual with chickenpox

Correct answer is: An unvaccinated 8-month-old with a chronic immunocompromising condition who came into close contact with an individual with active chickenpox. Immune globulin provides passive immunity for a comparatively short time, but the immunity is virtually immediate. Varicella zoster immune globulin (VZIG) is a pooled blood product and can be used to reduce the severity of chickenpox among select high-risk children, adults and pregnant women without evidence of varicella immunity. Vaccination or a history of chickenpox should confer immunity to varicella zoster. VZIG is not indicated for prevention of infection prior to any potential exposure (e.g., starting daycare)

Which of the following best describes the appropriate use of insulin lispro? In an insulin pump As a mealtime insulin As a basal insulin To prevent the Somogyi effect

Correct answer is: As a mealtime insulin. Insulin lispro is ultra-short-acting, with an onset 15-30 minutes after administration and a peak of 30 minutes to 2.5 hours. This insulin is typically used as a mealtime insulin. The American Diabetes Association (ADA) endorses the use of mealtime insulin to achieve glycemic control in patients whose A1c and post meal blood glucose remains elevated with the use of optimized therapy, particularly with basal insulin.

According to the American Heart Association, which of the following should be performed during a pre-participation sports physical examination? Routinely obtaining a resting 12-lead electrocardiogram Assessment of aortic span Auscultation of heart sounds in supine and standing positions Evaluation of jugular venous pressure

Correct answer is: Auscultation of heart sounds in supine and standing positions. The primary purpose of a sports pre-participation physical examination is to assess for risk of sudden cardiac death. In youth and young adults, hypertrophic cardiomyopathy is a risk that must be ruled out. As a result, auscultation of heart sounds in both supine and standing positions is necessary to assess for the murmur unique to this condition, which is paradoxically more apparent in the standing position.

You see an 8-year-old well child of Africian ancestry. Hemogram results are as follows: -Hg=10.0 g/dL (12-14 g/dL) -Hct=30% (36%-42%) -MCV=69 fL (80-96 fL) -RDW=13% (11%-15%) -RBC=5.4 million (3.2-4.3 million) These findings are most consistent with: Acute blood loss Beta thalassemia minor Iron deficiency anemia Cooley's anemia

Correct answer is: Beta thalassemia minor. Thalassemia is an anemia characterized by altered hemoglobin due to a genetically based variant - therefore, the mean cell volume (MCV) is typically quite low. Acute blood loss anemia would be characterized by a normal MCV. While iron deficiency anemia is microcytic, it is often accompanied by chronic blood loss, and the MCV is not as low as in thalassemia. Cooley's anemia is a form of thalassemia major, and the patient would not present as well.

You see an 18-year-old African American male with a BMI=34 kg/m2 who is newly-diagnosed with type 2 diabetes mellitus with an A1c=8.6%. Which of the following classes of medications is most commonly recommended as part of first-line therapy for this patient? DPP-4 inhibitor SGLT-2 inhibitor Thiazolidinedione Biguanide

Correct answer is: Biguanide. The vast majority of patients with type 2 diabetes mellitus are overweight or obese, and the primary underlying pathophysiology is peripheral cell insulin resistance. As a result, the first-line indicated therapy is a biguanide, whose primary mechanism of action is to increase insulin receptor sensitivity. Thiazolidinediones (TZD) are primary insulin receptor sensitizers that can be used as adjunct to the biguanides or as initial therapy when biguanides are contraindicated; recent safety concerns with TZD have led to infrequent use of this drug class. DPP-4 inhibitors increase the synthesis and release of insulin from beta cells and can be used in combination with biguanides if an adequate response is not achieved with biguanide therapy alone. SGLT2 inhibitors increase urinary glucose excretion and can be used in combination with one or more agents, but not as a first-line choice

A 66-year-old woman being managed for Addison's disease presents for follow-up evaluation. Findings consistent with an excessive dose of the medication taken for this condition would include: Diffuse hyperpigmentation. Blood pressure of 168/98 mm Hg. Loss of axillary hair. A white blood cell count of 6,000/mm3.

Correct answer is: Blood pressure of 168/98 mm Hg. The first-line medication for Addison's disease is prednisone; the goal is to replace endogenous cortisol in a manner that is consistent with the normal, physiologic diurnal variation. A typical starting dose is 15 mg q AM and 10 mg q PM. If the dose of prednisone is too high for this patient, she can demonstrate signs and symptoms of hypercortisolism, such as hypertension. Diffuse hyperpigmentation and loss of axillary hair are consistent with Addison's disease and will improve with medication. With respect to the white blood cell count, excess prednisone will result in leukocytosis and the count will be >12,000/mm3

Mrs. Perry presents her 11-month-old for evaluation of a 3-day history of crankiness and fever. Physical examination is significant for a temperature of 103.4⁰F (39.7⁰C) and bilateral, red, immobile tympanic membranes. The child has had a poor appetite for the past 2 days but has not vomited or had diarrhea. The child's chest is clear to auscultation. Additionally, the child is somewhat lethargic and does not age-appropriately resist the examination. Your next best action is to: Start the child on amoxicillin and ibuprofen Reassess in 1 hour after giving an antipyretic Order a CBC with WBC differential and blood culture Administer IM ceftriaxone

Correct answer is: CBC with WBC differential and blood culture. This child clearly has acute otitis media. The fact that he is showing signs of lethargy and does not resist the examination suggests that his illness is more complicated, possibly including hypoxemia, decreased cardiac output, and increased intracranial pressure and suggestive of disseminated infection. The child needs a full evaluation for sepsis, which includes a CBC with WBC differential, blood culture, urinalysis with urine culture, chest x-ray, and lumbar puncture for CSF analysis and culture. An evaluation for sepsis is usually started in the emergency department with inpatient admission to follow

Which of the following is the most appropriate first-line treatment for pelvic inflammatory disease in a 28-year-old otherwise well woman? Ceftriaxone IM plus doxycycline PO with or without metronidazole PO Ciprofloxacin plus metronidazole Trimethoprim plus sulfamethoxazole Ampicillin/sulbactam plus ofloxacin

Correct answer is: Ceftriaxone IM plus doxycycline PO with or without metronidazole PO . For most cases of PID, outpatient therapy with oral and/or parenteral antibiotics is sufficient. Ceftriaxone, 250 mg intramuscularly as a one-time dose, followed by doxycycline 100 mg PO bid for 2 weeks with or without metronidazole 500 mg PO is likely the most commonly used treatment regimen and is highly effective. The addition of metronidazole is helpful in the treatment of bacterial vaginosis that is often found in the woman with PID as well as provides activity against select anaerobes that are often part of this typically polymicrobial infection. As a result of the emergence of quinolone-resistant N. gonorrhoeae, regimens that include these agents are no longer routinely recommended

Which of the following is the most likely causative pathogen of bacterial vaginitis with cervicitis in a 22-year-old sexually-active woman? Escherichia coli Klebsiella pneumoniae Staphylococcus epidermidis Chlamydia trachomatis

Correct answer is: Chlamydia trachomatis. Vulvovaginitis is one of the most common gynecological problems and can result in symptoms consisting of vaginal discharge, itching, and pain. Chlamydial infection is most commonly diagnosed in young women between the ages of 18 and 35 years with a history of multiple sexual partners. Clinical presentation of cervicitis includes a friable cervix. Cervical motion and adnexal tenderness usually present with upper reproductive tract infection such as pelvic inflammatory disease. The causative organism, C. trachomatis, is an obligate intracellular bacteria closely related to Gram-negative bacteria. Treatment options for cervicitis caused by C. trachomatis include oral antimicrobials active against intracellular organisms, such as doxycycline, azithromycin (preferred agent due to efficacy with a single oral dose), and erythromycin.

A 47-year-old well woman presents with the following laboratory data: -HBsAg=present -AST=56 U/L (0-40 U/L) -ALT=98 U/L (0-40 U/L). These findings are most consistent with: Evidence of effective hepatitis B immunization. Evidence of hepatitis B infection in the past. Immunity against future hepatitis B infection. Chronic hepatitis B.

Correct answer is: Chronic hepatitis B. This patient is seropositive for the hepatitis B surface antigen (HBsAg present), which means that she is currently infected with the hepatitis B virus (HBV). Her AST and ALT are minimally elevated, within three times upper limit of normal, and she presents as a well woman. This presentation is inconsistent with acute HBV infection, but rather is consistent with chronic infection. The fact that she is indeed infected with hepatitis B virus precludes evidence of effective immunization.

Which of the following advice should you give to a breastfeeding mother whose 6-month-old infant has gastroenteritis who is mildly dehydrated? Limit breastfeeding to once or twice per day for the duration of the illness while using a diluted soy-based formula for additional feeding Give the infant oral rehydration solution only Continue breastfeeding as the baby tolerates Supplement breastfeeding with 1:1 diluted sports drink

Correct answer is: Continue breastfeeding as the baby tolerates. Breastmilk is probably better tolerated than any other food source in a 6-month-old infant in the setting of gastroenteritis. Oral rehydration, ginger ale, sports drink, or any other liquid will just as likely produce hypermotility and associated vomiting and/or diarrhea.

A 49-year-old female of European ancestry with type 2 diabetes mellitus was started on lisinopril 20 mg tablet daily 6 weeks ago for the management of hypertension. Today her blood pressure is 138/88 mm Hg and the patient is feeling well. The appropriate action at this time would be to: Order a white blood cell count to assess for neutropenia. Continue on her current medication regimen. Add HCTZ 12.5 mg to enhance HTN control. Assess renal function

Correct answer is: Continue on her current medication regimen. Lisinopril, an angiotensin-converting enzyme inhibitor, is the appropriate class of medication in the patient with diabetes and comorbid hypertension according to the Joint National Committee 8 (JNC-8) report. Given her age and DM, her blood pressure goal is <140/90 mm Hg. As she is at goal, current HTN treatment plan is adequate and can be continued. If in the future, HTN control is inadequate, her lisinopril dose can be increased and/or a second agent such as HCTZ can be added.

Mrs. Jansen is a 61-year-old patient who has difficult-to-manage type 2 diabetes mellitus. After trials of several oral medication combinations, the nurse practitioner added basal and mealtime insulin to the regimen 3 months ago. Today Mrs. Jansen's A1c is 6.8%. The appropriate response is to: Increase the insulin by 20% . Consider discontinuing one of the oral agents. Continue the present regimen. Repeat the A1c in one month.

Correct answer is: Continue the present regimen. The American Diabetes Association (ADA) published guidelines for the diagnosis and management of diabetes establishes the therapeutic goal of management as a A1c of <7%. At 6.8%, Mrs. Jansen has attained the therapeutic goal, and the appropriate response is to continue her present regimen. There is no indication to change any part of the plan, and as she is stable on this regimen, it is appropriate to repeat her A1c in three months.

In caring for an acutely symptomatic woman with lactation-associated mastitis, you advise her to: Continue to nurse only on the unaffected side Pump the breasts and discard the milk from the affected side Continue to nurse the baby as much as the mother tolerates Initiate a course of antimicrobials

Correct answer is: Continue to nurse the baby as much as the mother tolerates. An engorged breast can actually contribute to mastitis, so the breast should be emptied completely. It is safe for the baby to nurse when the mother has mastitis, and so normal breastfeeding patterns should not be interrupted. If the mother is uncomfortable, she can be encouraged to begin with the unaffected breast, then switching to the affected breast. If putting the baby on the affected breast is too painful, the breast should be emptied gently through manual or pumped milk expression.

Which of the following is a commonly encountered sign of allergic rhinitis? Dennie's lines Maxillary sinus pain Eczema Purulent nasal discharge

Correct answer is: Dennie's lines. Dennie's lines are exaggerated lines or folds in the skin below the lower eyelid that result from edema in the infraorbital lymphatic ducts that occurs in response to allergic inflammation. Maxillary sinus pain and purulent discharge are characteristic of an infectious disease, and eczema is another form of allergy that may or may not coexist with allergic rhinitis

The mother of a 6-year-old girl expresses concern over reports in the news that girls are reaching puberty at an earlier age. You explain that the most common reason for precocious puberty in girls is: Presence of a hormone-producing tumor Contribution of hormonal influence from an overactive pituitary gland Ingestion of exogenous estrogen or estrogen mimics present in the environment and a variety of foods Early onset of normal puberty

Correct answer is: Early onset of normal puberty. The average age of puberty onset, or transition from Tanner 1 to Tanner 2, is 8 to 13 years. Most females who experience precocious puberty (start of puberty between 6 and 8 years old) are otherwise healthy children whose pubertal maturation begins at the early end of the normal distribution curve. CNS imaging studies of these otherwise healthy children typically reveal no structural abnormalities. Expert consultation should be sought for the evaluation of the child with precocious puberty

When attempting to identify preschoolers at greatest risk for lead (Pb) poisoning, the most sensitive component of the assessment is the preschoolers: Environmental history Socio-demographics Hemoglobin level Developmental assessment

Correct answer is: Environmental history. The single biggest risk for lead poisoning is ingestion of lead-based paint. Therefore, identifying those who live in environments with lead exposure is critical to early screening and identification of lead exposure and poisoning. Lead-related abnormalities in physical examination, hemoglobin, and developmental progression are not apparent until long after lead poisoning has occurred.

The most likely causative organism in acute, uncomplicated UTI in nearly all patient groups is: Klebsiella species. Proteus mirabilis. Escherichia coli. Staphylococcus saprophyticus

Correct answer is: Escherichia coli. For acute, uncomplicated UTIs, the Gram-negative organism, Escherichia coli, is the predominant pathogen in nearly all patient groups. Other potential pathogens include the Gram-positive organisms Staphylococcus saprophyticus and enterococci. First-line treatment for acute, uncomplicated UTIs in women of reproductive age is oral trimethoprim-sulfamethoxazole when the local E. coli resistance rate is <20% and the patient does not have a sulfa allergy. If the local E. coli resistance rate is greater than 20%, or if the woman has a history of sulfa allergy, then the use of oral nitrofurantoin or fosfomycin is recommended. The use of ciprofloxacin should be limited given trends of increasing resistance by E. coli and better understanding ot fluoroquinolone adverse effects

You see a 24-year-old woman in her third trimester of pregnancy who is diagnosed with an uncomplicated UTI. She has not been exposed to any antimicrobials in the past 3 months. The most likely causative organism is: Klebsiella spp Enterococcus faecalis Escherichia coli Staphylococcus saprophyticus

Correct answer is: Escherichia coli. In both the pregnant and non-pregnant woman, E. coli is the most common cause of uncomplicated UTI. The organism is a common intestinal inhabitant and the close proximity of the anus and urethra puts women at risk for ascension of the organism into the lower urinary tract. In pregnancy, hormonal changes and dilation of the renal pelvis actually favor ascension into the upper urinary tract and, therefore, UTI is treated aggressively

Delayed gastric emptying is noted with the use of: Exenatide. Pioglitazone. Acarbose. Glyburide.

Correct answer is: Exenatide. Exenatide is a GLP-1 agonist, a drug class that is characterized by delayed gastric emptying and also includes liraglutide, lixisenatide and albiglutide. This is normally well tolerated unless the patient has other risk factors for gastroparesis. Pioglitazone is an insulin receptor sensitizer. Acarbose blocks gastric absorption of sucrose but does not impact gastric motility. Glyburide is an insulinogenic drug that impacts the beta cell of the pancreas and has no impact on the gut.

What is the strongest risk factor predisposing a child to the development of allergic rhinitis? Eczema Family history Large family size Elevated IgE

Correct answer is: Family history. Eczema and elevated IgE can be present, but they are not risk factors. Hypersensitivity has a clear hereditary component, and family history is an important risk factor for allergic conditions

Symptoms of gonococcal vaginitis typically include all of the following except: Purulent and/or blood-tinged vaginal discharge. Painful or burning urination. Vaginal bleeding during intercourse. Fever >100.5ᵒF (38.9ᵒC).

Correct answer is: Fever >100.5ᵒF (38.9ᵒC). Gonococcal vaginitis is caused by the Gram-negative diplococcus Neisseria gonorrhoeae, which is also a common cause of sexually transmitted disease. Most men with gonococcal infection do not present with any symptoms. For women, common symptoms include a milky to purulent and occasionally blood-tinged vaginal discharge, painful or burning urination, vaginal bleeding during intercourse, and lower abdominal pain during intercourse. When gonococcal infection is limited to the lower reproductive tract, fever is typically not noted.

A 5-year-old boy presents with reticular-form, maculopapular facial rash with a "slapped cheek" appearance. He has had a mild headache and myalgia for the past week and has a current temperature of 101.6⁰F (38.7⁰C). This presentation is most consistent with: Fifth disease Kawasaki disease Rubeola Scarlet fever

Correct answer is: Fifth disease. Erythema infectiosum (Fifth disease) is a common childhood viral exanthema caused by parvovirus. It is most common in children aged 4-7 years, and classically presents with the "slapped cheek" appearance of a maculopapular facial rash on the cheeks. The dermal manifestations fade over approximately 4 days. Occasionally systemic symptoms such as headache and myalgia occur. Rubeola is measles with a different trajectory, epidemiology, and cutaneous lesions - the classic "Koplik spots." Rubella, or German measles, has a more generalized macular rash and occurs more commonly in older children. Scarlet fever is the rash associated with strep throat.

You see a 27-year-old woman in her second trimester who is diagnosed with prenatal depression. In counseling her about the use of an SSRI during pregnancy, you consider that studies to date reveal that: In exposed neonates, increased rates of irritability have been reported The drugs have a negative effect on intellectual development A clear teratogenic pattern has been identified There is an increased rate of SIDS in exposed offspring

Correct answer is: In exposed neonates, increased rates of irritability have been reported. SSRIs have been widely-used during pregnancy as prenatal depression has been linked to postpartum depression. As more and more data become available about the effects of SSRI use during pregnancy, it has become apparent that there is an increased rate of irritability, though the effect is temporary. There is no link to impaired intellectual development or SIDS, and a clear teratogenic pattern has not been identified. However, paroxetine should be avoided during pregnancy as research suggests a small increase in the risk of fetal heart defects

A first-time father brings in his 1-month-old infant for evaluation. He is concerned that she "throws up all of the time," describing this as a small amount of slightly curdled-appearing liquid post each nursing or bottle feeding. The child has an appropriate weight gain since the last office visit at age 1 week, is alert, interactive, and appears content. The nurse practitioner knows that the most common cause of frequent spit-up and vomiting in the young infant is: GI tract immaturity allowing reflux Overfeeding Pyloric stenosis Allergy or intolerance to a component of infant formula

Correct answer is: GI tract immaturity allowing reflux. All organ systems must adjust to extrauterine life, and the gastrointestinal system is among the last to adapt. Frequent spitting up and reflux is very typical in the first months of life. As long as the infant is gaining weight and meeting developmental milestones, there is no cause for concern or additional diagnostics or therapeutics. Overfeeding would more likely manifest as inappropriate weight gain, pyloric stenosis would present as projectile vomiting with weight loss, and allergy will likely be accompanied by cutaneous rash and suboptimal weight gain.

When developing a management plan for a 58-year-old man with a 20-year history of type 2 diabetes mellitus, you recognize that which of the following oral medications is less likely to be effective in controlling plasma glucose because of his long-standing condition? Metformin Canagliflozin Glipizide Pioglitazone

Correct answer is: Glipizide. The pathophysiologic trajectory of type 2 diabetes mellitus is that early in disease the beta cells of the pancreas produce supraphysiologic levels of insulin as a compensatory response to the peripheral insulin resistance and subsequent decrease in effect. However, after several years of supraphysiologic production, the beta cell eventually burns out and cannot produce significant amounts of insulin release. An insulin stimulator, such as glipizide, will not be effective and exogenous insulin must be added to the regimen. Metformin, pioglitazone and canagliflozin (an SGLT2 inhibitor) do not stimulate the beta cell and their mechanisms of action are still likely to be effective.

Which of the following best describes a component of sitagliptin's mechanism of action? Increases hepatic glucose utilization Glucose-dependent insulin release Facilitates renal glucose excretion Diminishes glucose absorption in gastrointestinal tract

Correct answer is: Glucose-dependent insulin release. Sitagliptin is a dipeptidyl peptidase-4 (DPP-4) inhibitor. DPP-4 inhibitor slows inactivation of two incretin hormones, glucose-dependent insulinotropic polypeptide (GIP) and glucagon-like peptide-1 (GLP-1). This ultimately increases glucose-dependent insulin release and inhibits hepatic gluconeogenesis.

Which of the following is most consistent with findings in thyroid cancer? Hard, fixed mass of 2 cm in diameter "Hot spot" on thyroid scan Low TSH Presence of TPO antibodies

Correct answer is: Hard, fixed mass of 2 cm in diameter. Cancerous thyroid nodules tend to be non-productive "dead" tissue that does not produce thyroid hormone. As a result, cancerous nodules do not actively pump iodine from plasma, and an iodine-uptake scan will reveal this absence of activity as a "cold spot" of unproductive tissue. Conversely, toxic nodules that autonomously produce thyroxine will demonstrate hyperactivity or "hot nodules." Because thyroid cancer is a non-functioning tissue, thyroid hormone levels can fall resulting in a compensatory increase in thyroid stimulating hormone (TSH). Presence of TPO antibodies suggests autoimmune thyroid disease but not cancer. A hard, fixed mass larger than 1‒1.5 cm should raise suspicion of malignancy and be referred for fine-needle aspiration biopsy.

The dyslipidemia pattern most often seen in a person with untreated or poorly-controlled type 2 diabetes mellitus is one of: High TG and normal LDL-C and HDL-C. High TG, high LDL-C, and low HDL-C. Normal TG and high LDL-C and HDL-C. Low TG, low LDL-C, and high HDL-C.

Correct answer is: High TG, high LDL-C, and low HDL-C. Type 2 diabetes mellitus (DM) is an endocrine abnormality characterized by decreased storage of the metabolic fuel sources (glucose, amino acids, and triglycerides). As a result of decreased insulin activity there is an increase in unstored triglycerides, producing the characteristic hypertriglyceridemia of diabetes mellitus. Similarly, there can be decreased hepatic uptake of LDL-C resulting in elevated plasma LDL-C levels. The mechanism of low HDL-C is unclear but has been linked to insulin resistance

During a well-child examination, a 13-year-old boy states that he is the shortest in his class and wonders if he will ever catch up. Physical examination reveals sparse growth of long, slightly pigmented pubic hair in the genital area. The nurse practitioner counsels him that: His growth spurt will start soon His serum human growth hormone (HGH) level should be measured A CT scan of his growth plates should be performed He is likely near his adult height

Correct answer is: His growth spurt will start soon. The adolescent growth spurt does not occur until Tanner stage 3, so this patient has not reached that point in his physical development. Having reached Tanner stage 2 by the age of 13 (as demonstrated by the start of pubic hair growth) reassures the nurse practitioner that the patient's puberty is progressing appropriately. The best response is to tell him that he has not achieved his growth spurt yet, but that it will likely begin soon. A hormonal assessment or CT scan is not appropriate for this patient as his developmental pattern is consistent with normal expectations.

The most common cause of cervical squamous intraepithelial neoplasia is: Human papillomavirus. Bacterial infection. Herpes simplex virus. Parasitic organisms.

Correct answer is: Human papillomavirus. Human papillomaviruses are a collection of over 150 viruses, 40 of which are sexually transmitted. Of all of the sexually transmitted types, HPV-16 and HPV-18 have been implicated in approximately 70% of cervical neoplasms. The original FDA-approved quadrivalent HPV vaccine provided protection against HPV types 6, 11, 16 and 18. A nine-valent HPV vaccine is also available that provides protection against 5 additional types (31, 33, 45, 52, and 58). The 9-valent vaccine has the potential to prevent approximately 90% of cervical, vulvar, vaginal, and anal cancers.

When assessing of a 9-year-old boy with a Still's murmur, the NP recognizes that it is associated with a(n): Humming, vibratory quality Murmur that becomes louder with standing Defect in the mitral valve Indication for antiplatelet use to prevent thrombosis

Correct answer is: Humming, vibratory quality. A Still's murmur is found only in children and is a consequence of blood splashing into the aorta after ventricular contraction. This is a benign murmur with a characteristic humming, vibratory quality that becomes louder when the patient lies down due to intracardiac pressure. This murmur is not pathologic, and therefore its presence is not a reason to deny sports participation.

Which of the following characteristics applies to type 2 diabetes mellitus? Inactivity is a potent risk factor for the condition. Genetics is associated with type 1 but not type 2 diabetes mellitus risk. Exogenous insulin is consistently needed throughout the course of the disease. BMI has little to no impact on risk for this condition.

Correct answer is: Inactivity is a potent risk factor for the condition. Type 2 diabetes mellitus is a complex metabolic disorder characterized by a variety of risk factors including inactivity, obesity, genetics, medication, and aging. Obesity is more often abdominal and hyperinsulinemia is often noted in early disease; thus, exogenous insulin is typically not needed.

Mrs. Griffin is a 46-year-old woman with type 2 diabetes mellitus who is using metformin and a single 10 unit daily dose of the long-acting insulin, glargine. Her fasting blood glucose has been between 100-120 mg/dL (5.6-6.7 mmol/L). Which of the following best describes the next step in her therapy? Continue on the current glargine dose. Increase her glargine dose by 1 unit per day. Increase her glargine dose by 4 units per day. Increase her glargine dose by 6 units per day.

Correct answer is: Increase her glargine dose by 1 unit per day. The current approach to the management of type 2 diabetes mellitus is that when added, glargine insulin should begin at 10 units daily and then titrated up or down to reach a target fasting blood glucose of approximately 100 mg/dL. When fasting blood glucose remains 141-180 mg/dL, dose should be increased by 4 units. If the fasting glucose is 100-120 mg/dL, the dose should be increased by 1 unit, and when it is >180 mg/dL, the dose should be increased by 6 units

SGLT2 inhibitors work by: Increasing glucose utilization in the muscle. Reducing insulin resistance in the skeletal muscle and adipose tissue. Increasing urinary glucose excretion. Increasing pancreatic insulin release.

Correct answer is: Increasing urinary glucose excretion. SGLT2 inhibitors increase urinary glucose excretion and can be used in combination with one or more agents, but not as a first-line choice. These agents do not enhance insulin secretion or sensitivity. Use of these agents is associated with increased risk of genital candidiasis.

When counseling a breastfeeding woman about alcohol use during lactation, you inform the woman about all of the following except: Alcohol use inhibits the letdown reflex Alcohol use decreases milk production Infant intoxication can be seen with the mother ingesting as little as 5‒10 ounces of wine or its equivalent during a 2-hour time period Alcohol will readily pass into breast milk due to its low molecular weight

Correct answer is: Infant intoxication can be seen with the mother ingesting as little as 5‒10 ounces of wine or its equivalent during a 2-hour time period. Alcohol decreases milk production in the breast and inhibits the letdown reflex, and studies suggest that the presence of alcohol in breast milk causes the infant to ingest approximately 20% less. Infant intoxication is not seen with 5‒10 ounces of wine or its equivalent during a 2-hour time period, but infants exposed to alcohol can have impaired motor development and altered sleeping patterns. The molecular weight of alcohol is quite low at 46 Da and it will readily cross the mammary epithelial membrane to concentrate in breast milk.

A 5-year-old boy with asthma presents with his parents. He uses albuterol 2 puffs 3-4 times a day to control wheezing. You consider prescribing: No medication as he has adequate symptom control Oral theophylline Inhaled salmeterol Inhaled budesonid

Correct answer is: Inhaled budesonide. In accordance with National Asthma Education Prevention Program Expert Panel Report 3 (NAEPP EPR-3), the patient who requires albuterol several times a day should be on a daily anti-inflammatory agent. An inhaled corticosteroid (ICS) is the initial drug of choice for asthma patients who require daily anti-inflammatory medication.

A grandmother brings in her 6-year-old grandson who is diagnosed with streptococcal pharyngitis. You prescribe oral penicillin for the boy, but the grandmother asks if you still offer a "penicillin shot." You consider the following when counseling her about the use of IM penicillin: The oral version has a better spectrum of activity compared to the IM formulation Injectable penicillin for bacterial pharyngitis therapy offers no therapeutic advantage over an appropriately-used course of oral penicillin There is a lower risk of allergic reaction with the oral preparation compared to injectable penicillin Injectable penicillin is reserved for strains of streptococci exhibiting reduced susceptibility to the drug

Correct answer is: Injectable penicillin for bacterial pharyngitis therapy offers no therapeutic advantage over an appropriately-used course of oral penicillin. Penicillin is the drug of choice for a confirmed streptococcal infection. IM forms offer no therapeutic advantage over oral forms in terms of efficacy, spectrum of coverage, or risk of allergic reaction. Because it is always preferable to use the least invasive route of administration, the oral route is preferred.

A patient is currently taking combined oral contraception and now wants a progestin IUD. The first day of her last period was 10 days ago and she has not had sex since then. What is the best course of action? Insert the progestin IUD within 5 days of the next menstrual period. Insert the progestin IUD now but advise a barrier method of contraception for two weeks. Insert the progestin IUD now but continue COC for one week. Insert the progestin IUD today and advise that no additional method is needed.

Correct answer is: Insert the progestin IUD today and advise that no additional method is needed. The progestin IUD can be inserted and presumed to confer protection immediately. There is no requirement to wait until the next period and no reason to advise a back-up method. A pregnancy test is not needed given her recent sexual history.

The use of which of the following medication class has the potential for causing the greatest reduction in A1c for a 45-year-old man with a 7-year history of type 2 diabetes mellitus and a current A1c of 9.5%? SGLT2 inhibitor Thiazolidinedione DPP-4 inhibitor Insulin formulation

Correct answer is: Insulin formulation. Virtually all of the non-insulin agents used in the management of type 2 diabetes mellitus augment, either directly or indirectly, the production or action of insulin. As a result of both pharmacokinetic and pharmacodynamic properties, there is an end point which in the best case scenario is approximately 2% A1c reduction. Conversely insulin, an endogenous compound, can be progressively increased in dose until the desired A1c is achieved

During a well child visit of a 12-month-old girl, the parents mention that she has completely switched from formula to whole milk. She drinks about 4 cups (924 mL) each day, but eats few solids. A major concern of this diet for the child is an increased risk of: Vitamin B12 deficiency Iron deficiency anemia Lactose intolerance Celiac disease

Correct answer is: Iron deficiency anemia. In young children, iron deficiency anemia can occur as a result of the loss of iron stores from birth and change in diet. A 12-month-old who is drinking milk has lost the iron supplementation that came from either breast milk or formula and has lost her birth stores; eating few solids means that she is not getting much iron from food. The NP should encourage the parents to broaden the child's diet to include a variety of meats, fruits, vegetables, and cheese to reduce the risk of a dietary deficiency of iron.

When prescribing glargine, the clinician should consider that: Its mechanism of action differs from that of other insulins. It is used as basal insulin. It reaches peak effect at 4-8 hours after injection. It can be mixed in the same syringe with rapid-acting insulin.

Correct answer is: It is used as basal insulin. Glargine insulin is characterized by its steady-state impact on serum glucose over its entire duration of action. Glargine insulin does not have an identified peak, and is only used as basal insulin. Otherwise, its mechanism of action is like any other insulin; it binds to insulin receptors catalyzing intracellular movement of glucose and amino acids. Like any long-acting insulin, glargine insulin cannot be mixed in the same syringe with short-acting insulin preparations. Other insulins that are used as basal insulin include insulin detemir and newer ultra-long-acting insulins (e.g., insulin degludec, glargine U300).

Using metformin in a person with advanced renal impairment potentially increases the risk of: QTc prolongation. Renal failure. Poor glycemic control. Lactic acidosis

Correct answer is: Lactic acidosis. In an otherwise healthy patient, metformin does not typically cause significant adverse effect. However, in cases of impaired renal function, metformin is not readily eliminated and circulating metformin levels can elevate. This elevation in plasma metformin leads to an increase in metformin-mediated lactate production. Like metformin, lactate is not efficiently excreted in cases of decreased renal function. As a result, the elevated lactate production, exacerbated by inefficient excretion, results in elevated circulating lactate and subsequent lactic acidosis.

The nurse practitioner is examining a thriving 5½-month-old infant who was born 8 weeks premature. Which of the following is anticipated during a normal well-child examination? Rolls back to stomach Laughs Sits with support Hand-to-hand transfer

Correct answer is: Laughs. For the first 24 months of life the developmental expectations must be corrected for prematurity; a 5½-month-old infant born 8 weeks early, or at 32 weeks' gestation, would be expected to reach the developmental milestones of a 3½-month-old. Consequently, this infant would be expected to laugh as that is a normal milestone for a 3-month-old. Rolling back to stomach, hand-to-hand transfer, and sitting with support occur later

A 27-year-old woman presents with a 2-day history of fever, pelvic pain, vaginal discharge, and dysuria. Which of the following laboratory findings would support a diagnosis of pelvic inflammatory disease (PID)? Thrombocytopenia Leukocytosis with neutrophilia Elevated procalcitonin level Antinuclear antibody positive

Correct answer is: Leukocytosis with neutrophilia. PID (pelvic inflammatory disease) is an infectious disease consisting of endometritis, salpingitis, and oophoritis. Approximately 60% of infections are acquired through sexual transmission. Clinical presentation usually includes lower abdominal pain, abnormal vaginal discharge, dyspareunia, fever, gastrointestinal upset, or abnormal vaginal bleeding. PID should be considered when a women presents with new-onset lower abdominal or pelvic pain coupled with at least one of the following findings on clinical examination: cervical motion tenderness, uterine tenderness or adnexal tenderness. Supporting laboratory findings in PID include elevated erythrocyte sedimentation rate or C-reactive protein level and leukocytosis with neutrophilia.

Which of the following is most consistent with a hemogram finding in a 2-year-old with iron deficiency anemia? Elevated MCV, normal MCHC, NL RDW Low MCV, low MCHC, elevated RDW Low MCV, low MCHC, NL RDW Normal MCV, normal MCHC, elevated RDW

Correct answer is: Low MCV, low MCHC, elevated RDW. Iron deficiency anemia (IDA) is a condition of abnormal hemoglobin synthesis, so consequently the mean cell volume is low. Because it is an evolving problem over time, the average red blood cell distribution width (RDW) changes as new red blood cells are made with less iron than old red blood cells. Therefore, the MCV is low and the RDW is elevated in IDA.

Clinical presentation of a woman with ectopic pregnancy is likely to include: Uterine size greater than normal for gestational age Protracted amenorrhea Nausea and vomiting Low serum hCG level for gestational age

Correct answer is: Low serum hCG level for gestational age. An ectopic pregnancy occurs when the products of conception lodge outside the uterus, usually in a Fallopian tube. In some cases the conceptus expires and is biologically debrided. In others, the conceptus will continue to grow and divide until the physical constraints of the environment no longer allow it. In these cases, an adnexal mass is occasionally palpable. Uterine size is typically small for gestational age as the uterus is not participating in embryonic development. Protracted amenorrhea is not likely; indeed intermittent to constant menstrual-like bleeding is often seen in ectopic pregnancy. Finally, nausea and vomiting are not likely-when these symptoms occur in the first trimester, this is usually a result of normal hormonal surges that do not occur in an ectopic pregnancy. In ectopic pregnancy, serum hCG levels are low for gestational age (<6000 mIU/mL, with most cases having hCG levels <2000 mIU/mL)

Two weeks ago, the nurse practitioner saw a 2-year-old toddler in clinic. The child had not had a well-child check since he was 4 months old and received a number of "catch-up" vaccines at the visit. Today, his mother calls to report that he has a flat, pink rash "all over his body" but appears otherwise well. The nurse practitioner suspects that this is a normal reaction to which vaccine? Varicella MMR DTaP Injected influenza virus vaccine

Correct answer is: MMR. A diffuse macular rash is an anticipated adverse effect of the MMR vaccine. It is characterized by its general distribution and does not occur until two weeks after vaccination on average. While the varicella vaccine can produce a mild "chicken pox" type rash, it is not macular but rather blister-like. DTaP does not cause generalized rash, although it can cause a localized erythematous reaction. Injected influenza virus vaccine does not typically cause a rash, but can cause localized soreness at the injection site.

Which of the following symptoms is most consistent with a diagnosis of bacterial vaginosis? Fever Pain while urinating Malodorous vaginal discharge Abdominal pain

Correct answer is: Malodorous vaginal discharge. Bacterial vaginosis is the most common type of vaginitis in women of reproductive age. The condition occurs when certain species of normal vaginal bacteria undergo abnormal proliferation and cause inflammation. Most likely polymicrobial in nature, causative organisms likely include Gardnerella vaginalis and Mycoplasma hominis. The most common symptom is a malodorous vaginal discharge that can be worse after intercourse. The discharge is occasionally grayish, white, or yellow. However, a large proportion of women with bacterial vaginitis do not notice any symptoms.

A 50-year-old man who is obese has offered complaints that were consistent with classic gastroesophageal reflux disease. One week ago, the patient was started on a proton pump inhibitor, and given instructions that it needed to be taken at least 30 minutes before the morning meal. The patient appeared to understand and follow instructions. At follow-up the expected finding would be: Marked improvement in symptoms. Transient diarrhea. Mild nausea without vomiting. Headache

Correct answer is: Marked improvement in symptoms. Simple gastroesophageal reflux disease (GERD) responds well to proton pump inhibitor (PPI) therapy the large majority of the time; the key is in having the medication used properly. Proton pumps are at peak production during the cephalic and gastric phase of eating. PPIs will be most effective if therapeutic serum levels are achieved coincident with peak proton pump efficacy. Therefore, the medication should be taken at least 30 minutes before eating to allow for proper absorption and distribution. When a patient reports suboptimal response to PPI therapy for GERD, the first step is to ensure that the PPI is being taken at least 30 minutes prior to the morning meal

According to the National Asthma Education Prevention Program (NAEPP) Expert Panel Report-3 (EPR-3), the recommended strength of daily corticosteroid for use in the management of a 2-year-old with moderate persistent asthma is which of the following? Low-dose inhaled Medium-dose inhaled High-dose inhaled Low-dose oral

Correct answer is: Medium-dose inhaled. The recommendations of the expert panel report rely heavily on inhaled corticosteroids as the foundation for maintenance therapy in asthma. Management of moderate persistent asthma should always include a medium-dose inhaled corticosteroid, with additional measures, such as an inhaled long-acting beta-agonist, if necessary.

A 27-year-old female was evaluated one week ago for vaginal bleeding. She reported a history of normal menses until "a month or two ago." She is not sure when it started, but she had been spotting/bleeding almost continuously for the last 3-4 weeks. She is sexually active and is not using contraception. She denied abdominal pain, fever, or any other symptoms. Pelvic ultrasound two weeks ago revealed a gestational sac in the uterus but no heartbeat appreciated. A quantitative hCG was 15,000 IU/L. Today her repeat hCG is 49,000 IU/L. The nurse practitioner informs the patient that she: Likely has a molar pregnancy and needs additional radiographic testing Could have an ectopic pregnancy and needs to have another ultrasound today Appears to have had a spontaneous abortion and will need dilation and evacuation Most likely has an intrauterine pregnancy that will be confirmed by ultrasound in a few weeks

Correct answer is: Most likely has an intrauterine pregnancy that will be confirmed by ultrasound in a few weeks. There are a variety of reasons why early first trimester bleeding can occur; it is not always an indicator of spontaneous abortion or ectopic pregnancy. Given the irregular bleeding pattern, it is difficult to estimate the gestational age at the time of ultrasound. The marked rise in hCG indicates a normally-developing pregnancy, and is consistent with the gestational sac visualized in the uterus. The heartbeat of the fetus is usually present at about 6-weeks' gestation. The absence of fetal heartbeat in this patient clearly meant that the gestational age was <6 weeks at the time of ultrasound.

Developmental milestones expected during the well-child examination of a 5½-year-old child include: Using plurals Voicing an intended career Naming a best friend Drawing a person with ≥12 parts

Correct answer is: Naming a best friend. A child aged 5½ years has typically started school and this is when the concept of best friend emerges. Social interaction and relationships are new and, therefore, of paramount importance. Use of plural forms generally begins earlier, by the age of 4. Identifying an intended career and drawing a child with 12 or more parts comes after the age of 6.

While evaluating a 27-year-old woman with a 1-day history of dysuria, which of the following findings in urinalysis is most suggestive of urinary tract infection (UTI) caused by a Gram-negative organism? Nitrites 30 mg/dL protein Epithelial cells pH>8

Correct answer is: Nitrites. While protein, epithelial cells, and an elevated pH can be present in urinary tract infection, the presence of nitrites is most specific to Gram-negative bacterial infection. In the presence of Gram-negative bacteria, native nitrates are converted to nitrites in urine that has been retained in the bladder for at least 30 minutes.

All of the following are characteristics of an innocent murmur except: Occurs late in systole Does not radiate beyond the precordium Intensity increases with activity Becomes softer when going from supine to standing position

Correct answer is: Occurs late in systole. Innocent murmurs virtually always become softer when the patient moves from a supine to a standing position, as a consequence of decreased volume in the thoracic space. These murmurs are always softer than the normal S1 and S2 sounds, can be heard at any place in the precordium, and can occur at any time in systole.

Which of the following describes, in part, metformin's mechanism of action? Reduces insulin resistance Facilitates renal glucose excretion Stimulates insulin production Inactivates incretin

Correct answer is: Reduces insulin resistance. Metformin has a multimodal mechanism of action that includes (1) sensitization of peripheral insulin receptors and (2) decreasing the rate of hepatic gluconeogenesis. Metformin does not facilitate renal excretion, nor does it impact the incretin system.

Diagnostic evaluation of a person with irritable bowel syndrome (IBS) is most likely to reveal: Low hemoglobin level. Elevated ESR. Fecal occult blood. Tenderness in the sigmoid region.

Correct answer:Tenderness in the sigmoid region. A person with IBS usually has tenderness in the sigmoid region but the remainder of the examination is usually normal. Laboratory analysis is usually directed at ruling out another cause of the symptoms associated with the condition and will typically reveal a normal hemogram, a normal ESR, and a negative test result for the presence of fecal occult blood. Imaging studies of the GI tract are also usually normal

Ana is a 30-year-old female who had her third child 3 months ago. She has not had what she would call "a normal period" since giving birth, but she has not had any vaginal bleeding for almost two weeks. She wants to have a progestin implant (Nexplanon) inserted. A urine pregnancy test is negative, but she does report having had unprotected sex within the last two weeks. The best action would be to: Insert the implant today but advise Ana to use a backup method for one week. Tell Ana that she is not a candidate for the implant until she has had a normal menstrual cycle. Offer Ana another method of contraception and insert the implant within 5 days of the start of her next menstrual period. Suggest to Ana that a progestin IUD would be more appropriate if she wants insertion today.

Correct answer is: Offer Ana another method of contraception and insert the implant within 5 days of the start of her next menstrual period. Because Ana's last episode of vaginal bleeding was more than two weeks ago and she has had unprotected sex, she is not a candidate for implant insertion today. While she does not need to go through an entire "normal" cycle, the implant should not be inserted until she begins her next period. Therefore, the best advice is to suggest another method of contraception, such as the pill/patch/ring or barrier, and then schedule her for insertion of the implant within 5 days of the start of her next period. The guidelines for progestin IUD are the same as those for progestin implants, therefore a progestin IUD is not an appropriate alternative.

Louise is a 19-year-old G0P0 female who recently became sexually active with a male partner for the first time. She wants to have a copper IUD placed because her older sister has one and has been very happy with it. Her last menstrual period was 10 days ago, and her urine pregnancy test is negative. She says that her only unprotected sexual episode since her last period was last night. The nurse practitioner: Offers to insert the copper IUD today. Offers oral emergency contraception with IUD insertion to follow. Advises Louise to wait until her next period for copper IUD insertion. Suggests another form of birth control due to her age and nulliparity

Correct answer is: Offers to insert the copper IUD today. Since Louise's only episode of unprotected sex was less than 5 days ago, there is no reason to delay insertion of a copper IUD; in fact, the IUD will function as a form of emergency contraception to protect her from an unwanted pregnancy resulting from the unprotected encounter last night. While older forms of intrauterine devices were indicated only in women who have had at least one child, the most recent Practice Bulletin from the American College of Obstetricians and Gynecologists suggests that nulliparous women and adolescents can benefit from copper IUD use

For Megan in the previous question, first-line treatment for her IBD flare can include: Oral ciprofloxacin. Oral mesalamine. Adalimumab injection. Oral methotrexate.

Correct answer is: Oral mesalamine. In Crohn's disease and UC, oral aminosalicylates, including sulfasalazine (Azulfidine) and mesalamine (Apriso) are usually the first-line therapy. In UC, when disease is limited to the distal colon, mesalamine and corticosteroids can be administered rectally. Oral or parenteral corticosteroid use can provide rapid symptom relief because of potent antiinflammatory effects. In Crohn's disease, metronidazole and ciprofloxacin are used when perineal disease or an inflammatory mass is noted. However, antibiotic use in UC is discouraged because of the increased risk of C. difficile infection. Immune modulators including 6-mercaptopurine and azathioprine are often prescribed to provide long-term disease control. Other immune modulators such as methotrexate and cyclosporine have been used with some success

If the clinician hears a murmur that is believed to be consistent with hypertrophic cardiomyopathy (HCM) during a sports clearance physical exam in a 17 year-old basketball player, the clinician should do which of the following? Order an echocardiogram and allow sports participation pending the results of this evaluation Order an echocardiogram, consult with cardiology, and deny participation pending the results of the evaluation Order a 12-lead ECG and give full clearance for sports participation based on ECG results Order an exercise treadmill test and give full clearance for sports participation

Correct answer is: Order an echocardiogram, consult with cardiology, and deny participation pending the results of the evaluation. Hypertrophic cardiomyopathy is potentially quite dangerous and should be evaluated immediately by cardiology. Clearance should not be given for sports participation without evaluation by a cardiology specialist due to risk of sudden cardiac death with or immediately post exercise due to cardiac outflow tract blockade.

A 25-year-old G3, P2, Ab0 woman with gestational age of 37-weeks presents with painless vaginal bleeding, has soaked 3 pads in the past 2 hours. Examination reveals a soft, relaxed uterus consistent with gestational age. The most likely diagnosis is: Hemorrhagic cervicitis Placental abruption Membrane rupture Placenta previa

Correct answer is: Placenta previa. Placenta previa is among the most common causes of painless vaginal bleeding in the third trimester of pregnancy. When the placenta is anchored close to the cervix, uterine expansion that occurs late in pregnancy can stretch the placenta across the cervix and result in painless bleeding. Membrane rupture is not bloody, placenta abruption is not painless, and cervicitis typically represents an inflamed cervix that bleeds easily with any type of manipulation. The level of bleeding described in this question, in the absence of pain, is most consistent with placenta previa.

Developmental assessment of a healthy 11-month-old is expected to reveal an infant who: Walks solo Plays "peek-a-boo." Sits without support Can identify several body parts

Correct answer is: Plays "peek-a-boo." The infant has likely achieved several of these developmental milestones, but the best answer to the question is the milestone that is most closely associated with the reported age. An 11-month-old will likely have started sitting with support 3-5 months ago, and would have begun imitating "bye-bye" 2 months ago. The average child will not begin walking until after 12 months of age. However, at approximately 11 months, an infant is expected to begin playing "peek-a-boo."

A 15-year-old young woman with a 5-day history of vaginal itch presents with perineal excoriation, vaginal erythema, and white, clumping discharge. She is not sexually active and recently completed an antimicrobial course for acute bacterial rhinosinusitis. You expect to find the following on laboratory examination: Abundant white blood cells >20 clue cells/HPF Pseudohyphae Alkaline pH

Correct answer is: Pseudohyphae. The symptoms described are classic signs of a yeast infection - a fungal infection characterized by "spaghetti and meatball" pseudohyphae on microscopic examination. Abundant white blood cells suggest a bacterial or protozoan infection, clue cells are consistent with bacterial vaginosis, and an alkaline pH is suggestive of atrophic vaginitis that occurs as a consequence of decreased estrogen.

Which the following is found in a healthy-term neonate? Active gastrointestinal peristalsis Low stomach pH Relatively low serum albumin Well-developed hepatic metabolism

Correct answer is: Relatively low serum albumin. As survival inside the uterus requires different physiologic function as compared to extrauterine life, the neonatal period is characterized by a large spectrum of physiologic adaptation and change. Neonates normally have a comparatively low serum albumin that will increase significantly in the first weeks of life. This is the primary reason why protein-binding medications can offer such competition to bilirubin resulting in jaundice. Similarly, gastrointestinal function is immature in the neonate. Gastric pH is high because acid production is not mature. Gastrointestinal peristalsis is immature and the neonatal period can be characterized by frequent spitting up due to reflux of gastrointestinal contents after feeding. Finally, hepatic metabolism is immature at birth but matures rapidly in the first weeks and months of life

The use of which of the following antibiotics is most likely to reduce the effectiveness of combined oral contraceptive? Rifampin Amoxicillin Ciprofloxacin Doxycycline

Correct answer is: Rifampin. The coadministration of combined oral contraceptives and other medications that induce their metabolism can result in a circumstance in which oral contraceptive metabolism is accelerated. Rifampin is a cytochrome 3A4 inducer, and estrogen is a cytochrome 3A4 substrate. Administering these medications concomitantly risks accelerating metabolism of estrogen, and increases the risk of unwanted pregnancy. The use of penicillins, fluoroquinolones, and tetracycline antibiotics can result in increased breakthrough bleeding but without an observed increase in contraceptive failure

A 26-year-old well woman wants to begin injection contraception (medroxyprogesterone depot {Depo-Provera}) today. Her last menstrual period began 4 days ago. The nurse practitioner informs the patient that: She will have her first injection today and must return in 3 months for the next one. A urine pregnancy test is required and if the result is negative, she is able to have her first injection today. The first injection must be given on the first day of the menstrual period; she will need to use another method this month and return next month. Injection contraception is only appropriate today if she has not had unprotected sex in the last 5 days.

Correct answer is: She will have her first injection today and must return in 3 months for the next one. Because this patient's last period began less than 5 days ago, it is appropriate to begin hormonal contraception today. A pregnancy test is not required. The injection can be given, and the patient must be instructed to return in precisely 3 months for her next contraceptive injection so as not to experience an interruption in protection

A normal developmental milestone for a full-term infant at 3-5 months would include all of the following except: Sitting solo Reaching for objects Making a "raspberry" sound Recognizing food by sight

Correct answer is: Sitting solo. A 3-5-month-old infant will recognize food (e.g., the bottle or breast) by sight and will reach for it. An infant at this age is also capable of making a "raspberry" sound. Conversely, sitting solo is not mastered until 6-7 months.

Which of the following is a drug class that works predominantly by enhancing insulin release? Sulfonylureas Thiazolidinediones SGLT2 inhibitors Amylin analogues

Correct answer is: Sulfonylureas. The sulfonylureas are pure insulinogenic drugs. This drug class causes blocking of K+ channels on the membrane of the beta cell of the pancreas, resulting in membrane depolarization and calcium influx. The intracellular rise in calcium leads to insulin release. Sulfonylureas have no impact on insulin receptor activity, hepatic activity, or renal excretion.

Which is most likely to be reported in Graves' disease? Free T4=7.2 pmol/L (NL=10-27 pmol/L) TSH=0.09 mU/L (NL=0.15-4.0 mU/L) ESR=22 mm/h (NL <15 mm/h) TWBC=4,200/mm3, 0% Neutrophils w/ hypersegmentation (NL=6,000-10,000/mm3, 50%-70% Neutrophils)

Correct answer is: TSH=0.09 mU/L (NL=0.15-4.0 mU/L). Graves' disease is an autoimmune disorder characterized by the pathologic production of thyroid-stimulating immunoglobulins (TSI) that stimulate the thyroid gland and lead to elevated thyroxine levels. The elevated thyroxine levels suppress pituitary production of thyroid stimulating hormone (TSH). As a result, TSH levels will become very low, often undetectable. According to the American Thyroid Association (ATA), the TSH is the most sensitive indicator of thyroid function.

A 36-year-old man presents with a 12-h history of anorexia, nausea, and abdominal pain. Physical examination reveals positive obturator and psoas signs. Anticipated WBC with differential results are as follows: TWBC=5,200/mm3, Neutrophils=35%, Bands=1%, Lymphocytes=57% TWBC= 8,300/mm3, Neutrophils= 58 %, Bands=2%, Lymphocytes=40 % TWBC=6,800/mm3, Neutrophils=28%, Bands=3%, Lymphocytes=45%-55% with reactive forms TWBC=17,600/mm3, Neutrophils=64%, Bands=8%, Lymphocytes=24%

Correct answer is: TWBC=17,600/mm3, Neutrophils=64%, Bands=8%, Lymphocytes=24%. Positive obturator and psoas signs are both consistent with pain during peritoneal stretch in the region of the appendix and are closely associated with appendicitis; the coincident abdominal pain, anorexia, and nausea strengthen the diagnosis. Consequently, the white blood cell (WBC) count will likely demonstrate an increase characterized by elevated neutrophils and bands. A TWBC of 5,200/mm3 and lymphocytes of 57% is more consistent with viral infection. A total WBC of 8,300/mm3 with neutrophils=58%, Bands=2%, Lymphocytes=40% is within normal limits. However, the patient with a TWBC of 17,600/mm3 and neutrophils of 64% has an absolute neutrophil count of 11,264/mm3. This, along with the 8% bands, is highly suggestive of bacterial infection consistent with appendicitis.

Which of the following best describes the mechanism of action of the weight loss medication lorcaserin HCl (Belviq)? Increases elimination of glucose in the urine Targets receptors in hypothalamus to make the individual feel full sooner during a meal Prevents absorption of fat in the intestines Increases basal metabolic rate

Correct answer is: Targets receptors in hypothalamus to make the individual feel full sooner during a meal. Though the exact mechanism of action of lorcaserin (Belviq) is unknown, it is believed that the agent reduces food consumption and promotes earlier satiety by selectively activating the 5-HT2C receptors in the hypothalamus. Along with diet and exercise, lorcaserin can help overweight and obese patients with weight loss and maintaining weight loss.

The loss of the cremasteric reflex in a young man with a chief complaint of sudden-onset scrotal pain most likely represents: Testicular neoplasia. Acute epididymitis. Incarcerated hernia. Testicular torsion.

Correct answer is: Testicular torsion. Testicular torsion occurs when a testicle rotates, thus twisting the spermatic cord and cutting off the blood supply. The cremasteric reflex is a normal finding during physical examination and its absence is considered highly sensitive for testicular torsion to a 99% certainty. This superficial reflex, characterized by contraction of the cremaster muscle that pulls up the scrotum and testis on the side in response to light stroking of the ipsilateral inner thigh, remains present in other abnormalities of the testes, including neoplasm, epididymitis, and hernia. A scrotal ultrasound is used to confirm a diagnosis of testicular torsion, which will show reduced blood flow to the testicle. Prompt surgical correction is necessary to prevent testicular loss.

When selecting a treatment for gonococcal vaginitis, the nurse practitioner should consider which of the following? Resistance to fluoroquinolones is uncommon. The causative organism frequently produces beta-lactamase. Most women resolve the infection without antimicrobial treatment. Treatment duration is typically 10-14 days.

Correct answer is: The causative organism frequently produces beta-lactamase. When selecting treatment for gonococcal vaginitis, it is important consider that N. gonorrhoeae frequently produces beta-lactamase. Therefore, the choice of therapeutic agent should include agents with beta-lactamase stability, such as ceftriaxone and cefixime. Also, because of increasing rates of fluoroquinolone resistance by N. gonorrhoeae, the use of this class is no longer recommended for these infections. Recommendations from the Centers for Disease Control and Prevention recommend IM ceftriaxone as a single dose plus azithromycin as a single oral dose, which assumes concomitant C. trachomatis infection. Oral cefixime is an acceptable alternative to ceftriaxone only if injectable therapy cannot be used

In considering the use of a selective estrogen receptor modulator (SERM) for a 48-year-old woman, you consider all of the following except: Progestin opposition is not required. Osteoporosis risk is reduced. The frequency and intensity of hot flashes will be reduced. Use is not associated with an increased risk of breast cancer.

Correct answer is: The frequency and intensity of hot flashes will be reduced. Selective estrogen receptor modulators (SERMs; e.g., raloxifene [Evista]) are non-estrogen drugs that target very specific estrogen receptors and help prevent osteoclastic activity. They are not estrogen, and do not produce estrogen-mediated cancer risks; therefore, progestin opposition is not needed, and the use is not contraindicated in patients with a family history of breast cancer. Likewise, SERMs do not attenuate all estrogen-mediated symptoms, so hot flashes are not reduced.

Andrea Wilson is a 62-year-old woman who was seen a few weeks ago for evaluation of ongoing abdominal discomfort and nausea. Laboratory assessment revealed H. pylori and she was treated for peptic ulcer disease with an appropriate antibiotic/proton-pump inhibitor combination. Today she has completed her course of therapy and admits to no real change in symptoms. This suggests that: Treatment was inadequate and a salvage regimen should be used. It is too soon to expect a response and the patient should be reevaluated in four weeks. The patient could have complicated peptic ulcer disease and should be referred for upper endoscopy. Peptic ulcer disease is likely not the correct diagnosis.

Correct answer is: The patient could have complicated peptic ulcer disease and should be referred for endoscopy. Peptic ulcer disease includes both duodenal ulcers and gastric ulcers. Uncomplicated duodenal ulcers are the more common type and usually respond quickly to the appropriate therapeutic regimen. Presuming the diagnosis of peptic ulcer disease was correct, the absence of a response warrants endoscopic evaluation to rule out complicated disease, such as malignancy.

A 67-year-old woman is diagnosed with hypothyroidism and requires levothyroxine therapy. When considering initiation of levothyroxine therapy for this patient, the NP realizes that: The therapeutic dose should be lower than what is used in a younger patient. The elderly should have a rapid initiation of thyroxine therapy. Goal of therapy should be a thyroid stimulating hormone (TSH) of 5‒10 mIU/L. TSH levels should be checked 2 weeks after initiation of therapy

Correct answer is: The therapeutic dose should be lower than what is used in a younger patient. Thyroid hormone sets the metabolic rate for virtually all metabolic processes. Thyroxine excess can present as accelerated function in all body systems. As a function of age-related change, the cardiovascular system is particularly sensitive to any deviation from homeostasis, and thyroxine excess will often present as palpitations, dysrhythmia, and angina. In order to avoid symptoms of excess, thyroid hormone replacement should be introduced at a low level and increased gradually, with the ultimate goal of symptom control and a TSH 0.5-2.0 µg/mL. The TSH level should be checked after approximately 8 weeks of treatment. Since the elderly have slower basal metabolism and slower rate of thyroxine elimination, a lower dose will be required as compared to that in young and middle-aged adults.

Which of the following statements regarding the management of eczema in children is false? Topical immunomodulators are preferred over topical high-potency corticosteroid therapy when eczema occurs on the face Eczema is a manifestation of a type I hypersensitivity reaction Topical calcineurin inhibitors are not recommended for use in younger children (<12 years of age) Use of topical immunomodulators are associated with skin burning and pruritus

Correct answer is: Topical calcineurin inhibitors are not recommended for use in children (<12 years of age). Eczema is common in young children and is a result of a type I hypersensitivity reaction. Consistent use of medium- or high-potency topical steroids can cause lipodystrophy and secondary fungal infection. Steroid-sparing treatments are preferred when eczema occurs on the face, groin, or axillary areas. Topical calcineurin inhibitors (e.g., tacrolimus, pimecrolimus) are immunomodulators that have been shown to be effective in treating eczema in children. Use of these agents are associated with skin burning and pruritus.

The results of a radioactive iodine uptake scan of a 52-year-old woman with a thyroid mass reveal a cold spot. This finding is most consistent with: Amiodarone-induced thyroid disease. Thyroid cyst. Metabolically active thyroid nodule. Graves' disease.

Correct answer is:Thyroid cyst. A thyroid cyst is a non-functional collection of tissue on the thyroid gland. Non-functional tissue does not uptake iodine, and on a radioactive iodine uptake scan a cyst will appear as non-functional or "cold." Amiodarone-induced thyroid disease, active thyroid nodule, and Graves' disease are all conditions of increased gland function, and nodules would scan as hyperfunctioning or "hot."

A 26-year-old nonpregnant woman presents with uncomplicated UTI. She is otherwise healthy, has not received any systemic antimicrobials in the past year, does not have a sulfa allergy, and is not taking any medications other than combined oral contraceptives for birth control. The local E. coli resistance rate to TMP/SMX is about 10%. The preferred oral therapy for this patient is: Trimethoprim-sulfamethoxazole. Amoxicillin. Nitrofurantoin. Cephalexin.

Correct answer is: Trimethoprim-sulfamethoxazole. Escherichia coli is the most common cause of acute, uncomplicated urinary tract infections in women. First-line treatment is oral trimethoprim-sulfamethoxazole when the local E. coli resistance rate is <20% and the patient does not have a sulfa allergy. If the local E. coli resistance rate is greater than 20%, or if the patient has a history of sulfa allergy, then the use of oral nitrofurantoin or fosfomycin is recommended. The use of ciprofloxacin should be limited given trends of increasing resistance by E. coli

A 24-year-old woman who is diagnosed with polycystic ovary syndrome (PCOS) is at greater risk of developing: Type 1 diabetes mellitus. Type 2 diabetes mellitus. Addison's disease. Hypothyroidism.

Correct answer is: Type 2 diabetes mellitus. Insulin resistance is now recognized as a root cause (along with genetics and other factors) of polycystic ovary syndrome (PCOS). When patients are insulin resistant, the body perceives this as an insulin deficiency and the pancreas compensate by making more insulin, resulting in hyperinsulinemia. For several years, the elevated insulin levels will compensate for the insulin resistance, and the patient remains normoglycemic. However, the hyperinsulinemia can also trigger increased production and release of androgens, which in turn leads to the development of ovarian cysts and impaired fertility, in part due to anovulation. If hyperinsulinemia and insulin resistance is left untreated for several years, type 2 diabetes mellitus will likely develop.

Which of the following is the likely trigger of Kawasaki syndrome? Unknown Hormonal changes Environmental exposure to a toxin Viral

Correct answer is: Unknown. Kawasaki syndrome is a systemic vasculitis of unknown etiology. In this disease, the proposed pathophysiology of this disease is an immune-mediated response to some infectious trigger in patients with a certain genetic predisposition. The trigger is as of yet unidentified and believed not to be contagious from person-to-person. Treatment consists of specialty evaluation and care including high-dose aspirin and immune globulin

During the well examination of a healthy 4-year-old, the nurse practitioner expects the child's mother to report that the child is able to: Walk down the stairs alternating steps Ride a tricycle Verbalize what to do when cold, hungry, or tired Tie shoelaces

Correct answer is: Verbalize what to do when cold, hungry, or tired. The 4-year-old child has likely achieved several of these developmental milestones, but the best answer to the question is the milestone that is most closely associated with the reported age. Riding a tricycle and walking down alternating steps are expected of a 3-year-old. The fine motor skill of tying shoelaces typically occurs after age 6. However, a 4-year-old will have recently mastered the ability to identify the proper solution to basic immediate needs

A drug demonstrated to cause teratogenic effects and its use is not indicated in any circumstances is assigned risk category: B C D X

Correct answer is: X. The five pregnancy categories for drugs are A, B, C, D, and X. Category A drugs are those with demonstrated safety and only those therapeutic agents that represent normal physiologic replacement are assigned category A (e.g., insulin, thyroid hormone replacement, and vitamins in physiologic doses). Category B drugs are those that have no demonstrated harm in animal studies but no human study is available. Category C drugs have shown adverse effects in animal studies but there are no adequate, well-controlled studies in humans; potential benefits can warrant use in pregnant women. Category D drugs have demonstrated teratogenic effects in human but benefits can outweigh risk in life-threatening situations. Category X drugs are those that have demonstrated teratogenic effects and use is not indicated in any circumstances. Isotretinoin and the HMG-CoA reductase inhibitors (i.e., statins) are in this category

Which of the following women is the best candidate for progestin-only pill (POP) use? A 26-year-old with multiple sexual partners A 32-year-old who is breastfeeding a 3-week-old infant A 22-year-old who frequently forgets to take prescribed medications 35-year-old currently taking anticonvulsant therapy

Correct answer is:A 32-year-old who is breastfeeding a 3-week-old infant. Progestin-only pills (POP) are generally regarded as the better choice in breastfeeding mothers as they do not interfere with the milk supply in the way that estrogen-containing contraceptives can. POPs can safely be used during breastfeeding. However, progestin is metabolized within 20 hours and, therefore, needs to be taken quite consistently. Women who frequently forget to take their pills or tend to take medication at a different time every day are not good candidates. While women with multiple sexual partners can benefit from progestin compounds that thicken the cervical mucus, hormonal contraception is not a primary mechanism of preventing sexually transmitted infections. Though the use of POP with anticonvulsant therapy is not harmful to women, the interaction is likely to reduce the effectiveness of POPs

Which of the following is most consistent with the hepatic enzyme profile of a person with non-alcoholic fatty liver disease? AST=1208 U/L, ALT=560 U/L AST=45 U/L, ALT=88 U/L AST=678 U/L, ALT=990 U/L AST=98 U/L, ALT=149 U/

Correct answer is:AST=98 U/L, ALT=149 U/L. Non-alcoholic fatty liver disease is the most common cause of transaminase elevation and will produce a mild elevation in transaminases consistent with liver strain or liver cell (hepatocyte) damage anywhere from 1 to 4 times the upper limit of normal (ULN). As in most cases of liver insult, the AST and ALT are both elevated, with the ALT being the higher of the two and the ratio of AST to ALT less than 1. Fatty liver disease does not cause the profound transaminase elevation seen in other types of liver insult.

Megan is a 24-year-old woman diagnosed with inflammatory bowel disease (IBD). In the past 4 months, she has reported intermittent abdominal pain, diarrhea, weight loss, and fatigue. She now presents after 2 days of worsening symptoms. Which of the following laboratory findings is consistent with an IBD flare? A1c >7.5% Erythrocyte sedimentation rate (ESR)=14 mm/hr WBC <10,000 mm3 C-reactive protein (CRP)=15 mg/L

Correct answer: C-reactive protein (CRP)=15 mg/L. During an IBD flare, serological markers of inflammation, including C-reactive protein (CRP) and erythrocyte sedimentation rate (ESR) are usually elevated. Leukocytosis is also often present. The inflammation induced during an IBD flare should not have a clinically significant impact on blood glucose levels or A1c values in the absence of diabetes mellitus

Nancy is a 22-year-old African-American female who initially presented with significant thyrotoxicosis. She was subsequently diagnosed with Graves' disease. She was then referred to endocrinology and ultimately treated with ablation of the gland. Follow-up laboratory assessment would be expected to reveal: Absence of thyroid-stimulating antibodies. An increase in TSH as compared to pretreatment. An increase in free T3 as compared to pretreatment. Absence of thyroid peroxidase (TPO) antibodies.

Correct answer is:An increase in TSH as compared to pretreatment. Graves' disease is characterized by biologically active thyroid-stimulating antibodies that stimulate the thyroid gland and suppress endogenous TSH. As a result, pretreatment TSH is very low and sometimes undetectable. Following radioactive ablation of the gland, thyroxine output falls, circulating levels drop, and the anterior pituitary increases TSH production in an attempt to stimulate thyroxine release. Thyroid-stimulating antibodies can still be present, but the gland can no longer respond to them. T3 will not increase as the ablated gland cannot produce adequate levels of hormone. TPO antibodies are not characteristic of either treated or untreated Graves' disease; they are a marker for Hashimoto's thyroiditis.

A 26-year-old woman who is taking a combined oral contraceptive (COC) expresses interest in pregnancy. She asks you how long after discontinuing COC use will she be able to conceive. You respond: Immediately. After 1-2 months. After 3-4 months. After 5-6 months.

Correct answer is:Immediately. Combined oral contraception use, once discontinued, is not a barrier to safe conception. The hormones administered in contraceptives are metabolized and excreted within 24 hours, which is why consistent daily administration is so critical. This also applies for women who discontinue the use of a contraceptive patch or vaginal ring

A patient with Hashimoto's thyroiditis was started on levothyroxine 25 mcg/day. Eight weeks later, the nurse practitioner expects that the patient will report: A modest weight gain. Less fatigue. A decrease in palpitations. Longer sleeping hours.

Correct answer is:Less fatigue. Hashimoto's thyroiditis is an autoimmune disorder characterized by the production of thyroid peroxidase (TPO) antibodies that block thyroid-stimulating hormone (TSH) receptors on the thyroid gland and thus block the action of TSH. In the first months of disease, there will be a fluctuation in symptoms as the body attempts to compensate, but eventually the patient will become symptomatic of hypothyroidism. Replacing thyroid hormone is expected to improve symptoms of hypothyroidism. As a result, it is expected that the patient will report a decrease in the fatigue that is characteristic of thyroid hypofunction. A decrease in palpitations and sleep hours is more consistent with appropriate management of thyrotoxicosis, not hypothyroidism.

You see a 26-year-old woman diagnosed with bacterial vaginosis. She states she does not like to use vaginal products and asks for oral therapy. The appropriate oral treatment option is: Metronidazole. Ciprofloxacin. Cefixime. Fluconazole.

Correct answer is:Metronidazole. Bacterial vaginosis occurs when certain species of normal vaginal bacteria undergo abnormal proliferation and cause inflammation. Most likely polymicrobial in nature, causative organisms likely include Gardnerella vaginalis and Mycoplasma hominis. According to the Sanford Guide to Antimicrobial Therapy, treatment of bacterial vaginosis can include oral metronidazole or metronidazole vaginal gel, tinidazole, or clindamycin vaginal gel. Oral clindamycin can be used as an alternative agent. In pregnancy, the preferred treatment is oral metronidazole or oral clindamycin for 7 days

A 28-year-old man presents with a firm, round, painless ulcer in the genital area with localized lymphadenopathy that has been present for 2 weeks. He feels otherwise well. This most likely represents: Genital warts (Condyloma acuminata). Primary syphilis. Secondary syphilis. Genital herpes.

Correct answer is:Primary syphilis. Chancre is a key clinical finding in primary syphilis, presenting as a firm, round, painless genital and/or anal ulcer(s) with a clean base and indurated margins. This is usually accompanied by localized lymphadenopathy that lasts about 3 weeks in duration. Secondary syphilis is characterized by nonpruritic skin rash, often involving the palms and soles. Genital warts are characterized by verruca-form lesions, while genital herpes can involve painful ulcerated lesions. A test to detect T. pallidum directly from lesion exudate or tissue is the definitive method to diagnose primary syphilis. A presumptive diagnosis requires the use of two tests: a nontreponemal test (e.g., Rapid Plasma Reagent) and a treponomal test (e.g., fluorescent treponemal antibody absorbed test).

A 62-year-old woman with newly-diagnosed hyperthyroidism complains of periods of tremors and tachycardia. You recommend which of the following as a means to alleviate these symptoms? Clonazepam Propranolol Methyldopa Amlodipine

Correct answer is:Propranolol. Hyperthyroidism results in an increased metabolic rate in virtually all body systems. Tachycardia is a result of excess beta1-receptor stimulation in the heart, and tremor is associated with excess beta2-receptor stimulation in the central nervous system. Propranolol is a non-cardioselective beta-adrenergic antagonist, which blocks both beta1- and beta2-receptors. This non-selective blockade attenuates many symptoms of hyperthyroidism, including tachycardia and tremor. Once the hyperthyroidism is resolved, its associated symptoms should resolve and beta blocker treatment can be tapered and eventually discontinued

Treatment of Graves' disease involves elimination of overactive thyroid tissue via: Surgical removal. Laser ablation. Radioactive iodine. Electrodissection.

Correct answer is:Radioactive iodine. The thyroid gland's primary function is to actively pump in serum iodine. Virtually all circulating iodine is actively taken up into the thyroid gland. Radioactive iodine, when administered, essentially takes advantage of this normal function to infiltrate the gland with radioactive material. The radioactive material destroys the tissue, and the gland is ablated and rendered non-functional.

Which of the following is most consistent with untreated hypothyroidism? TSH <0.15 mIU/L (0.4-4.0 mIU/L), free T4=79 pmol/L (10-27 pmol/L) TSH=8.9 mIU/L (0.4-4.0 mIU/L), free T4=15 pmol/L (10-27 pmol/L) TSH=1.9 mIU/L (0.4-4.0 mIU/L), free T4=22 pmol/L (10-27 pmol/L) TSH=24 mIU/L (0.4-4.0 mIU/L), free T4=3 pmol/L (10-27 pmol/L)

Correct answer is:TSH=24 mIU/L (0.4-4.0 mIU/L), free T4=3 pmol/L (10-27 pmol/L). Hypothyroidism is a condition characterized by decreased circulating thyroid hormone with resultant elevated thyroid stimulating hormone (TSH). When circulating thyroid hormone levels are low, the pituitary gland will respond by releasing TSH. In cases of untreated hypothyroidism, it is expected that the TSH level will be elevated, and free T4 level will be low.

Match each type of emergency contraception with its proposed mechanism of action. A. Copper-IUD B. Ulipristal C. Levonorgestrel 1. Induces a sterile inflammatory response that is toxic to sperm and ova __ 2. Causes endometrial changes that alter likelihood of fertilized egg being implanted __ 3. Inhibits or delays ovulation __ 1-B 2-A 3-C 1-C 2-B 3-A 1-A 2-B 3-C 1-A 2-C 3-B

Correct answer: Copper-IUD- Induces a sterile inflammatory response that is toxic to sperm and ova; Ulipristal- Induces a sterile inflammatory response that is toxic to sperm and ova. Levonorgestrel- Inhibits or delays ovulation; Emergency hormonal contraception can have a variety of actions depending upon the contraceptive type and timing of its administration during the ovulation cycle. Levonorgestrel primarily works by inhibiting or delaying ovulation, but can also inhibit tubal transport of the egg or sperm. Ulipristal has a direct inhibitory effect on follicular development and ovum release. Additionally, changes in the endometrium with ulipristal use can diminish the likelihood of a fertilized egg being implanted. The presence of the copper-IUD in the intrauterine cavity induces a foreign body effect that results in a sterile inflammatory response that is toxic via a number of mechanisms to sperm and ova.

A 52-year-old man presents with fever, diarrhea, and left lower quadrant abdominal pain. Laboratory analysis reveals a WBC=18,000 mm3 with neutrophilia. Which of the following diagnostic procedures would be most useful to confirm a diagnosis of acute colonic diverticulitis? Barium enema Abdominal ultrasound Endoscopic evaluation Abdominal CT scan with contrast

Correct answer:Abdominal CT scan with contrast. In acute colonic diverticulitis, the diverticula are inflamed, causing fever, leukocytosis, diarrhea, and left lower quadrant abdominal pain. Intestinal perforation is the likely origin of the condition. Imaging is often obtained to support the diagnosis and assess disease severity or complications. An abdominal CT scan with contrast is helpful in identifying findings consistent with the condition inducing bowel wall thickening; complications including abscess and fistulas can also be identified. Because of the potential risk of complication, a barium enema should not be obtained during an acute episode of diverticular disease. Abdominal ultrasound is not helpful in this condition. Endoscopic evaluation of the colon is contraindicated in acute diverticulitis, as insufflation of air can result in or exacerbate free perforation and peritonitis

A 37-year-old man presents complaining of recurrent abdominal pain that is relieved with defecation. Symptom onset is often accompanied by bloating and a change in stool frequency and form. He denies seeing blood in the stool. The most likely diagnosis is: Irritable bowel syndrome. Ileus. Peptic ulcer disease. Ulcerative colitis.

Correct answer:Appearance of blood in the stool. Irritable bowel syndrome (IBS) is a functional GI disorder characterized by abdominal pain and altered bowel habits in the absence of specific and unique organic pathology. The Rome III criteria for the diagnosis of IBS require that patients must have recurrent abdominal pain with two or more of the following: discomfort relieved by defecation, symptom onset associated with a change in stool frequency, or symptom onset associated with a change in stool form or appearance. Additional symptoms usually include altered stool frequency, form, or passage (or a combination of two or all three) usually accompanied by mucorrhea and abdominal bloating or the sensation of distention or both.

Which of the following signs is most consistent with a diagnosis of Crohn's disease involving the small intestines? Chronic, dry cough Vomiting Constipation Blood in the stool

Correct answer:Blood in the stool. Inflammatory bowel disease (IBD) is a disease of unclear etiology, but likely involves an autoimmune response to the GI tract. The two major types of IBD are ulcerative colitis (UC, characterized by pathological changes limited to the colon) and Crohn's disease (changes can involve any part of the GI tract). The inflammation that occurs in Crohn's disease causes cells in the affected areas of the intestines to secrete large amounts of water and salt, which cannot be completely reabsorbed. The manifestations of IBD generally depend on the area of the intestinal tract involved. Patients with Crohn's disease involving the small intestine frequently have abdominal pain and cramping, reduced appetite and unintended weight loss, blood in the stool, and diarrhea

A nurse practitioner is seeing a 37-year-old male in follow-up and wants to evaluate the efficacy of treatment for a patient who was diagnosed with repeat H. pylori infection. Appropriate diagnostic testing includes: Serologic H. pylori antibodies. Biopsy via endoscopy. Serum gastrin level. Fecal antigen assay.

Correct answer:Fecal antigen assay. Evaluation for repeat infection cannot include antibody assessment as antibodies will remain indefinitely following initial infection. While biopsy via endoscopy would provide an accurate assessment, it is an invasive and expensive procedure and not necessary solely to diagnose repeat infection. A fecal antigen assay involves only the noninvasive collection of a stool sample and will discriminate recurrent, acute infection with the presence of antigen. A breath test can also be considered as a test for cure. Serum gastrin levels are not a marker for bacterial infection.

Risk factors for non-alcoholic fatty liver disease include all of the following except: Obesity. Hyperthyroidism. Type 2 diabetes mellitus. Hypertriglyceridemia.

Correct answer:Hyperthyroidism. Non-alcoholic fatty liver disease (NAFLD) described the condition of an accumulation of fat in the liver of persons who drink little or no alcohol. For most people, the condition causes no signs and symptoms. Risk factors include high cholesterol, elevated levels of triglycerides, metabolic syndrome, obesity, polycystic ovary syndrome (PCOS), and type 2 diabetes mellitus; these are all conditions associated with increased insulin resistance. Additional NAFLD risk factors include hypothyroidism, and hypopituitarism. NAFLD treatment includes minimizing the contributing risks.

You see a 46-year-old woman who states that she "got stomach flu that has been going around at work." She reports nausea, vomiting, and diarrhea over the past 18 hours. You consider all of the following except: Initiating antiemetic therapy. Counseling on staying hydrated. Prescribing antimicrobial therapy prophylactically. Counseling that symptoms should begin to resolve in the next day or two.

Correct answer:Prescribing antimicrobial therapy prophylactically. Cases of acute gastroenteritis can be caused by a number of organisms. Most commonly, this is a self-limiting viral infection that can spread quickly through contaminated food, water, or the environment. Less commonly, bacteria or parasites are the causative organism. Signs of gastroenteritis can include nausea, vomiting, diarrhea, and a low-grade fever. Usually symptoms peak within 24 to 48 hours of infection and resolve after 3 to 5 days. Patients should stay hydrated with clear liquids and an antiemetic can be considered to stop vomiting. Antimicrobial therapy should not be used routinely unless the causative organism is identified and therapy is deemed appropriate.

Oral antimicrobials commonly used to treat H. pylori gastrointestinal tract infection include all of the following except: Metronidazole. Vancomycin. Clarithromycin. Amoxicillin.

Correct answer:Vancomycin. Helicobacter pylori, a Gram-negative spiral-shaped organism, is found in at least 90% of patients with duodenal ulcer and 40%-70% of individuals with gastric ulcer. Eradication of the organism dramatically reduces the risk of a relapse and several antimicrobial combinations can be used to effectively treat symptomatic H. pylori infection. Amoxicillin and metronidazole are effective agents and H. pylori resistance to these agents is uncommon. Clarithromycin can also be used though resistance to this agent is increasing. Vancomycin is used to treat infections caused by Gram-positive organisms and is usually given parenterally (except for the treatment of C. difficile infection).

What are cullens sign and grey turners sign? What do they indicate?

Cullens Sign - edema and bruising of subQ tissue around the umbilicus Grey's Turner Sign - brusing/bluish discoloration of flank area that may indicate retroperitoneal bleed Can both indicate pancreatitis

Definition of IgM Anti-HAV

Hep A Antibody IgM Acute infection, patient is contagious, no immunity yet

Definition of HbeAg

Hepatitis B "e" antigen active viral replication, highly infectious persistence of "e" indicates chronic hep B

Definition of Anti-HBs

Hepatitis B Surface Antibody pt is immune either from past infection or vaccine

Definition of HBsAg

Hepatitis B Surface Antigen Screening test for Hep B if +, patient has virus and is infectious Indicates with acute infection or chronic hep b

A 12-year-old boy presents with acute bacterial rhinosinusitis. He has a beta-lactam allergy and had taken clarithromycin 2 months ago for a "bad cold." You recommend treatment with: Ceftriaxone Azithromycin Amoxicillin-clavulanate Levofloxacin

Levofloxacin . Common pathogens that cause acute bacterial rhinosinusitis (ABRS) in children include S. pneumoniae, H. influenzae, and M. catarrhalis. According to guidelines from the Infectious Diseases Society of America (IDSA), first-line treatment of acute bacterial rhinosinusitis in the patient with a beta-lactam allergy can include levofloxacin monotherapy or clindamycin plus cefixime or cefpodoxime. For this patient with a beta-lactam allergy, the use of amoxicillin would not be appropriate. His recent use of clarithromycin places him at risk for a macrolide-resistant infection (e.g., resistance to azithromycin). Levofloxacin would be an appropriate choice for this patient. Though levofloxacin is not indicated for patients <18 years of age for sinusitis, IDSA does recommend this agent for younger patients as an appropriate choice in certain situations. Ceftriaxone is not a recommended option for the treatment of acute bacterial rhinosinusitis in children.

Function of hypothalamus

coordinates nervous and endocrine system by sending signals via pituitary gland

Metformin max dose ?

Metformin max dose - 1000mg BID

Who are TCDs contraindicated in?

NYHA Class 3 and Class 4 heart disease, symptomatic heart failure

Action of rapid acting insulins

Onset 15 min, Peak 30 minutes to 2.5 hrs, Duration 4.5 hrs

Psoas sign/Iliopsoas sx

RLQ pain during 90 degree flexion of hip, pt should push against resistance and straighten leg

Lab results subclinical hyperthyroidism

TSH <0.5, normal T4, normal T3

What is subclinical hypothyroidism?

TSH >5 but normal free T4, question whether to treat to recheck in 6 mths

McBurney's Point

area located between iliac crest and umbilicus in RLQ

What doe elevated GGT indicate?

can be elevated in liver disease and biliary obstruction "Lone" elevation is sensitive indicator of possible alcoholism

Function of biguanides

decrease glucoeogenesis and decreases peripheral insulin resistance

Rovsing's Sx

deep palpation of LLQ which causes referred pain to RLQ (diverticulitis)

What is charcot's foot and ankle?

deformity of foot that is caused by joint and bone dislocation and fractures d.t neuropathy and loss of sensation

Function of Thiazolidinediones (TZDs)

enhances insulin sensitivity in muscle tissue, reduces hepatic glucagon production

Definition of IgG Anti-HAV

hep A antibody IgG type Positive indicates immunity no virus present, not infectious Hx of native hep a or vaccination

Lab findings primary hypothyroid

high TSH, low free T4, low T3

What is ulcerative colitis?

inflammatory bowel disease that affects colon/rectum, "squeezing cramping" pain left lower quadrant

What is crohns disease?

inflammatory bowel disease that may affect ANY part of GI tract, may have fistula formation and anal disease

Obturator sx

inward rotation of hip causes RLQ pain

Normal ALT level, indication?

normal 0-40 more specific for liver inflammation

Action of NPH

onset 1 hr, peak 6-14, duration 24 hrs

Action of short acting regular human insulin

onset 30 minutes, peak 1-5 hrs, duration 6-8 hrs

Action of premixed 70/30, 50/50

onset 30 minutes, peak 4.5 hrs, duration 24 hrs

Action of basal insulin

onset 30 minutes, peak none, duration 24 hrs

Murphy's maneuver

press deeply into RUQ under costal border

Markel Test

raise heels and drop down suddenly , can also jump in place

Function of pituitary gland

stimuated by hypothalamus into producing stimulating hormones such as GSH, LH, TSH, ACTH, GH

Function of sulfonylureas

stimulates beta cells of pancreas to secrete more insulin

Lab findings for Graves Disease

very low TSH with elevated free T4 and T3, will have positive TRAb (thyrotropin receptor antibodies, aka thyrostimulating immunoglobulin)


Set pelajaran terkait

Section Quiz 5-5 - Staff and Support Agencies

View Set

Chapter 3: Muscular Strength and Endurance

View Set

Independent living chapter 12 review

View Set